Difference between revisions of "2006 AMC 10A Problems/Problem 1"

 
 
(9 intermediate revisions by 7 users not shown)
Line 1: Line 1:
== Problem ==
+
#redirect [[2006 AMC 12A Problems/Problem 1]]
Sandwiches at Joe's Fast Food cost $3 each and sodas cost $2 each.  How many dollars will it cost to purchase 5 sandwiches and 8 sodas?
 
 
 
<math> \mathrm{(A) \ } 31\qquad \mathrm{(B) \ } 32\qquad \mathrm{(C) \ } 33\qquad \mathrm{(D) \ } 34\qquad \mathrm{(E) \ } 35 </math>
 
== Solution ==
 
== See Also ==
 
*[[2006 AMC 10A Problems]]
 

Latest revision as of 00:00, 28 April 2008